Search found 35 matches


In the question, "How many machines of Type R were used?" do specify "Type R" to throw you off? The problem already says the same number of each type of machine was used so "Type R" and "Type S" should be the same, right? Essentially isn't this question asking...

by coffee5251

Fri Feb 06, 2009 1:23 pm
Forum: Problem Solving
Topic: Rate
Replies: 7
Views: 2096

I also go 60%. What is the OA?

by coffee5251

Mon Feb 02, 2009 10:47 am
Forum: Problem Solving
Topic: How to solve this question
Replies: 11
Views: 15575

mals24 wrote:krisraam is right

Only if your denominator is in the form of 2^n, 5^n or 2^n*5^n then only will the number be a terminating decimal.
Is this true for every number or just for the problem above?

by coffee5251

Mon Feb 02, 2009 7:06 am
Forum: Problem Solving
Topic: terminating decimal
Replies: 14
Views: 3284

mals24 wrote:krisraam is right

Only if your denominator is in the form of 2^n, 5^n or 2^n*5^n then only will the number be a terminating decimal.
Is this true for every number or just for the problem above?

by coffee5251

Mon Feb 02, 2009 6:54 am
Forum: Problem Solving
Topic: terminating decimal
Replies: 14
Views: 3284

Re: Just trying

x + 2y + z = 8 2x + y + z = 7 These two equations tell us that y-x = 1 Substituting for y in the first eq...we have x + 2(1+x) + z = 8 3x + z = 6 ------ (A) Now from (I), we have x,y and z are positive integers. Since x and z are positive, z will be > 0. For equation A to be true X cannot have a va...

by coffee5251

Fri Jan 30, 2009 3:37 pm
Forum: Data Sufficiency
Topic: Equations
Replies: 5
Views: 1490

Re: HCF /LCM

nehakhas1 wrote:1. The HCF of three numbers is 12. Their product is 17280. What are the three numbers?
I. One of the numbers is greater than the other two.
II. One of the numbers is lesser than the other two.

Please solve
Out of curiosity does anyone know what level a problem like this is??

by coffee5251

Fri Jan 30, 2009 7:15 am
Forum: Data Sufficiency
Topic: HCF /LCM
Replies: 7
Views: 2934

Re: Norman window

A norman window consists of a retangle surmounted by a semicircle. Find the area of the norman window. How much wood fram is needed to enclose the window? L-6 W-4 I got the rectangle part of the proble... A=LW A=6*4=24 24+2Pi=30.28 How do I get the semicircle? Why do you add 2Pi to the area? How do...

by coffee5251

Thu Jan 29, 2009 5:26 pm
Forum: Problem Solving
Topic: Norman window
Replies: 4
Views: 5740

You could also test the answer choices. Take each answer choice (the current price of the towel) and divide it by 120. Then take each one again, add 1 for the price increase of $1 and then divide that number by 120. Then, see which two numbers have a difference of 10. for example, a: 120/1 = 120. th...

by coffee5251

Wed Jan 28, 2009 5:55 pm
Forum: Problem Solving
Topic: Word Trans (Alg Trans): A store currently charges the same p
Replies: 5
Views: 6571

DanaJ wrote:This means that s/4 = 30 or that s = 120.

IMO answer is D.
Do you mean E (120)?

by coffee5251

Wed Jan 28, 2009 4:36 pm
Forum: Problem Solving
Topic: PS tough question !
Replies: 4
Views: 2520

for such questions, why do we have to assume that all integers will be consecutive integers? why can't there be a repetition of some odd integers? the answer should be E. i don't understand how we've assumed that the integers between x and y are consecutive when this hasn't been mentioned in the qu...

by coffee5251

Fri Jan 23, 2009 4:46 pm
Forum: Data Sufficiency
Topic: Odd and Even
Replies: 7
Views: 2743

b. x<0, so that -x = 3x - 2. For this one you would have 4x = 2, with x =1/2. But 1/2 is not a negative, which means that the given equation has only one possible answer: x = 1. So I'd go with B. Hi, what do you mean when you say the given equation has only one possible answer : x=1? What does this...

by coffee5251

Fri Jan 23, 2009 3:40 pm
Forum: Data Sufficiency
Topic: tricky one
Replies: 8
Views: 1995

Re: Another number properties question

2x3x5x7 is not an integer together 2x2x3x5x7 = NO i 2x2x3x3x5x5x7x7 = YES Can you explain this? What do you mean by "2x3x5x7 is not an integer"? Also, if it says "k is divisible by exactly 4 prime numbers" does that mean they don't have to be distinct? (i.e. 2x2x3x3x5x5x7x7 is d...

by coffee5251

Fri Jan 16, 2009 6:54 am
Forum: Data Sufficiency
Topic: Another number properties question
Replies: 11
Views: 2390

Re: Another number properties question

2x3x5x7 is not an integer together 2x2x3x5x7 = NO i 2x2x3x3x5x5x7x7 = YES Can you explain this? What do you mean by "2x3x5x7 is not an integer"? Also, if it says "k is divisible by exactly 4 prime numbers" does that mean they don't have to be distinct? (i.e. 2x2x3x3x5x5x7x7 is d...

by coffee5251

Fri Jan 16, 2009 6:51 am
Forum: Data Sufficiency
Topic: Another number properties question
Replies: 11
Views: 2390

When you see a question like this, think factors find factors of 450 = 2 x 3 x 3 x 5 x 5 We know that 450Y = n^3 which means Y should atleast have 2 x 2 x 3 x 5 X N where N is of the form A^3 where A is an integer Y = 2^2 x 3 x 5 x A^3 Why is Y= 2^2 x 3 x 5 x A^3 if the prime factorization of 450 o...

by coffee5251

Fri Jan 16, 2009 5:22 am
Forum: Problem Solving
Topic: 450y - how to approach
Replies: 12
Views: 4795

Re: Problem on Multiples

(2) Fewer than 2 of the first 5 positive integer multiples of m are multiples of 12. m 2m 3m 4m 5m Fever than 2 = 1 so that would be m m is a multiple of 12 so it is a multiple of 6 Choose B Hi, maybe I'm not understanding the question, but why does m have to be a multiple of 12? What if m is 6, th...

by coffee5251

Thu Jan 15, 2009 11:50 am
Forum: Data Sufficiency
Topic: Problem on Multiples
Replies: 8
Views: 1783